Đến nội dung

PhanThai0301 nội dung

Có 166 mục bởi PhanThai0301 (Tìm giới hạn từ 25-05-2020)



Sắp theo                Sắp xếp  

#709390 [TOPIC] ÔN THI BẤT ĐẲNG THỨC $\boxed{\text{THPT CHUYÊN}}$...

Đã gửi bởi PhanThai0301 on 27-05-2018 - 20:09 trong Tài liệu - Đề thi

Mình có cách khác hay hơn cho bài này :icon6:

$<=>\frac{a^2}{a\sqrt{a^2+8bc}}+\frac{b^2}{b\sqrt{b^2+8ac}}+\frac{c^2}{c\sqrt{c^2+8ba}}$

$\geq \frac{(a+b+c)^{2}}{{a\sqrt{a^2+8bc}}+{b\sqrt{b^2+8ac}}+{c\sqrt{c^2+8ba}}}$ ($bunhiacovxki$)

Đặt $M={a\sqrt{a^2+8bc}}+{b\sqrt{b^2+8ac}}+{c\sqrt{c^2+8ba}}$

$={\sqrt{a}.\sqrt{a}\sqrt{a^2+8bc}}+{\sqrt{b}.\sqrt{b}\sqrt{b^2+8ac}}+{\sqrt{c}.\sqrt{c}\sqrt{c^2+8ba}}$

${\sqrt{a}\sqrt{a^3+8bca}}+{\sqrt{b}\sqrt{b^3+8bca}}+{\sqrt{c}\sqrt{c^3+8bca}}$

$M^2\leq (a+b+c)(a^3+b^3+c^3+24abc)=(a+b+c)(a^3+b^3+c^3+3.(2\sqrt{a}\sqrt{b}).(2\sqrt{b}\sqrt{c}).(2\sqrt{c}\sqrt{a}))$ ( $bunhiacovxki$)

$\leq (a+b+c)(a^3+b^3+c^3+3(a+b)(b+c)(c+a))$ ( $cauchy$)

$=(a+b+c)(a+b+c)^{3}=(a+b+c)^{4}$

$=> M \leq (a+b+c)^2$

$A\geq \frac{(a+b+c)^2}{(a+b+c)^2}=1$

$Q.E.D$

  Bạn ơi nếu a, b, c =0 thì tất cả cm của bạn đều sụp đổ, cái này đã có nhiều người nhầm lẫn.




#709377 $(1+a+b+c)(1+ab+bc+ca)\geq 4\sqrt{2(a+bc)(b+ca)(c+ab)...

Đã gửi bởi PhanThai0301 on 27-05-2018 - 18:26 trong Bất đẳng thức và cực trị

Cho a, b, c là các số dương. Chứng minh rằng:

        $(1+a+b+c)(1+ab+bc+ca)\geq 4\sqrt{2(a+bc)(b+ca)(c+ab)}$.




#709375 $\frac{a^{2}+b^{2}+c^{2}}{ab+bc+ca}+\frac{1}{3}\geq...

Đã gửi bởi PhanThai0301 on 27-05-2018 - 18:20 trong Bất đẳng thức và cực trị

Cho a, b, c là các số thực dương tùy ý. Chứng minh rằng:

   $\frac{a^{2}+b^{2}+c^{2}}{ab+bc+ca}+\frac{1}{3}\geq \frac{8}{9}(\frac{a}{b+c}+\frac{b}{c+a}+\frac{c}{a+b})$.




#709374 [TOPIC] ÔN THI BẤT ĐẲNG THỨC $\boxed{\text{THPT CHUYÊN}}$...

Đã gửi bởi PhanThai0301 on 27-05-2018 - 18:09 trong Tài liệu - Đề thi

BÀI 134

CMR: 

A=$\frac{a}{\sqrt{a^2+8bc}}+\frac{b}{\sqrt{b^2+8ac}}+\frac{c}{\sqrt{c^2+8ba}}\geq 1$

  Cách 1: Sử dụng BĐT Holder cách này đã trình bày nhiểu trên diễn dàn.

  Cách 2: Ta chọn số dương r sao cho:

                     $\frac{a}{\sqrt{a^{2}+bc}}\geq \frac{a^{2r}}{a^{2r}+2(bc)^{r}}$

                BĐT tương đương với $a^{2}(a^{2r}+2(bc)^{r})^{2}\geq (a^{2}+bc)a^{4r}$.

                                           <=> $b^{2r}c^{2r}+a^{2r}b^{r}c^{r}\geq 2a^{4r-2}bc$.   

                Áp dụng BĐT AM-GM ta có:

                          $b^{2r}c^{2r}+a^{2r}b^{r}c^{r}\geq 2a^{r}b^{\frac{3r}{2}}c^{\frac{3r}{2}}$.

                Để dấu "=" thì $r=4r-2;3r=2$ <=> $r=\frac{2}{3}$.

                Áp dụng BĐT AM-GM ta có:

                          $A\geq \frac{a^{2r}+b^{2r}+c^{2r}}{a^{2r}+b^{2r}+c^{2r}}=1$ (Q.E.D)

               Dấu bằng xảy ra khi <=> a = b = c.




#709301 $\sqrt{a-1}+\sqrt{b-1}+\sqrt{c-1} \leq \sqrt{a+...

Đã gửi bởi PhanThai0301 on 26-05-2018 - 18:11 trong Bất đẳng thức và cực trị

Help me

Cho a,b,c>1 và 1/a+1/b+1/c=2

$\sqrt{a-1}+\sqrt{b-1}+\sqrt{c-1}\leq \sqrt{a+b+c}$

(Olympic Iran 98)

 Vì $\frac{1}{x}+\frac{1}{y}+\frac{1}{z}=2$ => $\frac{x-1}{x}+\frac{y-1}{y}+\frac{z-1}{z}=1$.

 Áp dụng BĐT Cauchy - Schwarz ta có:

    $x+y+z\doteq (x+y+z)(\frac{x-1}{x}+\frac{y-1}{y}+\frac{z-1}{z})\geq (\sqrt{x-1}+\sqrt{y-1}+\sqrt{z-1})^{2}$.

 => đpcm.

 Dấu bằng "=" xảy ra khi x = y = z = 3/2.
 




#709270 $(a+b-c)^{2}(b+c-a)^{2}(c+a-b)^{2}\ge...

Đã gửi bởi PhanThai0301 on 26-05-2018 - 10:36 trong Bất đẳng thức và cực trị

Cho a, b, c là các số thực tùy ý. CMR $(a+b-c)^{2}(b+c-a)^{2}(c+a-b)^{2}\geq (a^{2}+b^{2}-c^{2})(b^{2}+c^{2}-a^{2})(c^{2}+a^{2}-b^{2})$.




#709239 $\boxed{\text{Chuyên Đề}}$ Bất đẳng thức - Cực trị

Đã gửi bởi PhanThai0301 on 25-05-2018 - 16:06 trong Bất đẳng thức và cực trị

Cho x,y,z>0 thỏa mãn 2x + 4y + 7z = 2xyz

Tìm GTNN: P=x + y +z

  Bài này mình đã post lên vmf mà chưa có ai giải bây giờ mình mới nghĩ ra xin trình bày lời giải như sau:

 

 Giả sử P đạt min khi x =a, y =b, z =c. Khi đó a, b, c >0 và 2a + 4b + 7c = 2abc.                         (1)

 Dễ thấy khi P đạt min thì $\frac{x}{a}=\frac{y}{b}=\frac{z}{c}=1$ và ta có thể viết các biểu thức x + y +z, 2x + 4y + 7z lại thành:

             $2x+4y+7z=2a.\frac{x}{a}+4b.\frac{y}{b}+7c.\frac{z}{c}$

 Áp dụng AM-GM ta có:

            $(x+y+z)^{2}(2x+4y+7z)\geq (a+b+c)^{2}(2a+4b+7c)[(\frac{x}{a})^{a}(\frac{y}{b})^{b}(\frac{z}{c})^{c}]^{\frac{2}{a+b+c}}[(\frac{x}{a})^{2a}(\frac{y}{b})^{4b}(\frac{z}{c})^{7c}]^{\frac{1}{2a+4b+7c}}$.

 Cái ta cần là 1 đánh giá dạng $(x+y+z)^{2}(2x+4y+7z)\geq kxyz$ để có thể sử dụng GT kaf suy ra kết quả bài toán.    Do đó, ta phải chọn các số x, y, z thích hợp sao cho số mũ của chúng bằng 1, tức là:

           $\left\{\begin{matrix} \frac{2a}{a+b+c}+\frac{2a}{2a+4b+7c}=1& \\ & \\ \frac{2b}{a+b+c}+\frac{4b}{2a+4b+7c}=1 & \frac{2c}{a+b+c}+\frac{7c}{2a+4b+7c}=1 \end{matrix}\right.$                          (2)

  Từ (1); (2) ta tìm đc $a=3,b=\frac{5}{2},c=2$. Lúc này ta có:

           $x+y+z\geq \frac{15}{2}$.

  Vậy Min x + y + z là $\frac{15}{2}$ <=> $x=3,y=\frac{5}{2},z=2.$




#709231 [TOPIC] $\text{Luyện đề ôn thi} $ $\boxed{\text...

Đã gửi bởi PhanThai0301 on 25-05-2018 - 12:27 trong Tài liệu - Đề thi

Câu 5: (1 điểm)
    Chứng minh với mọi số nguyên $m \times n \geq 3$ bao giờ cũng xây dựng được 1 bảng chữ nhật gồm $m \times n$ số chính phương đôi một khác nhau sao cho tổng mỗi dòng, tổng mỗi cột là một số chính phương.  
        

    Ta xét n số thỏa mãn $a_{1}=x_{1}^{2}$    là số cp chẵn ,$a_{2}=x_{2}^{2}<...<a_{m-1}=x_{n-1}^{2}$ là các số cp chẵn lớn hơn $a_{1}$ và nếu tổng $a_{1}+a_{2}+...+a_{n-1}=2k+1$ ta chọn $a_{m}=x_{n}^{2}=k^{2}$.

   Ta chọn dãy $b_{1}$ là số cp lẻ và $b_{1}\geq 3^{2}$.

   Chọn $b_{2}=y_{2}^{2},b_{3}=y_{3}^{2},...,b_{n-1}=y_{y-1}^{2}$ là các số cp chẵn thỏa mãn $y_{2}>x_{m}.y_{1},y_{3}>x_{m}.y_{2},...,y_{n-1}>x_{m}.y_{y-2}$ nếu $b_{1}+b_{2}+...+b_{n-1}=2k+1$ , chọn

$b_{n}=y_{n}^{2}=h^{2}$. Khi đó $b_{1}+b_{2}+...+b_{n}$ là số cp.

   Xét bảng m.n. Ta c/m đc bảng xây dựng như vậy là đúng vs đề bài.

   Xét dòng thứ i, tổng các số ở dòng i là $x_{i}^{2}y_{j}^{2}+...+x_{i}^{2}y_{n}^{2}=x_{2}^{2}(y_{j}^{2}+...+y_{n}^{2})$ là số cp.

    Tương tự tổng ở cột j là số cp.

     Mặt khác các số trong bảng phân biệt.

     => đpcm.

   




#709149 Cho $x, y, z> 0$ thỏa mãn $x+y+z= 2019$

Đã gửi bởi PhanThai0301 on 23-05-2018 - 21:31 trong Bất đẳng thức và cực trị

Cách khác của mình :

        Áp dụng BĐT Cauchy - Schwarz ta có:

               $\frac{x}{x+\sqrt{(x+y)(y+z)}}\leq \frac{x}{2x+\sqrt{yz}}$.

        Đặt $a=\sqrt{\frac{yz}{x^{2}}},b=\sqrt{\frac{xy}{z^{2}}},c=\sqrt{\frac{xz}{y^{2}}}$ thì abc = 1.

        Do đó ta cần chứng minh nếu abc = 1 thì

              $\frac{1}{2+a}+\frac{1}{2+b}+\frac{1}{2+c}\leq 1$.

        Quy đồng mẫu số rồi áp dụng BĐT AM - GM => đpcm.




#709131 ĐỀ THI THPT CHUYÊN TỈNH HƯNG YÊN NĂM HỌC 2018-2019

Đã gửi bởi PhanThai0301 on 23-05-2018 - 19:18 trong Tài liệu - Đề thi

Đề thi THPT chuyên tỉnh Hưng Yên vòng 2.

33407496_1955407044472293_90890300726307




#709130 Tìm GTNN của biểu thức P = x + y +z.

Đã gửi bởi PhanThai0301 on 23-05-2018 - 18:59 trong Bất đẳng thức và cực trị

Cho các số thực dương x, y, z thỏa mãn 2x + 4y + 7z = 2xyz. Tìm giá trị nhỏ nhất của biểu thức P = x + y + z.

                               




#709115 ĐỀ THI THPT CHUYÊN TỈNH HƯNG YÊN NĂM HỌC 2018-2019

Đã gửi bởi PhanThai0301 on 23-05-2018 - 14:49 trong Tài liệu - Đề thi

Đề thi vào chuyên tỉnh Hưng Yên vòng 1.

33096375_1667130760072289_64268341594439

P/s: nguồn lượm trên facebook.




#708930 [TOPIC] ÔN THI BẤT ĐẲNG THỨC $\boxed{\text{THPT CHUYÊN}}$...

Đã gửi bởi PhanThai0301 on 21-05-2018 - 16:29 trong Tài liệu - Đề thi

Bài 131: CMR với a, b, c dương thỏa mãn a + b + c = 3 thì

                         $\frac{a^{2}b}{2a+b}+\frac{b^{2}a}{2b+c}+\frac{c^{2}a}{2c+a}\leq 1$.

Bài 132: Cho a, b, c là các số thực tùy ý. Chứng minh rằng:

                        $\frac{ab}{c^{2}}+\frac{bc}{a^{2}}+\frac{ca}{b^{2}}\geq \frac{1}{2}(\frac{a+b}{c}+\frac{b+c}{a}+\frac{c+a}{b})$.




#708732 [TOPIC]: ĐA THỨC THCS

Đã gửi bởi PhanThai0301 on 19-05-2018 - 06:13 trong Tài liệu - Đề thi

 Tiếp tục nhá: các bài toán về xác định đa thức

 Bài 20: Tìm số nguyên m sao cho đa thức (x + m)(x - 3) + 7 phân tích được tthành (x + b)(x + a) với a, b là các số nguyên.

 Bài 21: Xác định đa thức bậc 3 biết f(0) = 1, f(1) = 0, f(2) = 5, f(3) = 22.

 Bài 22: Cho đa thức P(x) bậc 4 thỏa mãn P(-1) = 0 và P(x) - P(x-1)= x(x+1)(2x+1).

           1. Xác định P(x).

           2. Tính S= 1.2.3+ 2.3.5+ ...+ n(n+1)(2n+1).

 Bài 23: Xác định đa thức bậc 2 P(x) có 3 nghiệm phân biệt P(a)= P(b)= P(c)= 0.

 Bài 24: Đa thức f(x) khi chia cho x+1 dư 4, khi chia cho $x^{2}+1$ dư 2x + 3. Tìm đa  thức dư khi chia f(x) cho $(x+1)(x^{2}+1)$.

 Bài 25: Xác định đa thức f(x) có tất cả các hệ số đêu là số nguyên không âm nhỏ hơn 8 thỏa mãn: f(8)= 2003.

 Bài 26: Tìm tất cả các đa thức P(x) thỏa mãn đồng nhất thức: xP(x - 1)= (x - 2)P(x) với mọi số thực x.

 Bài 27: Tìm tất cả các đa thức P(x) thỏa mãn đồng nhất thức: (x - 1)P(x + 1)- (x + 2)P(x)= 0, với mọi số thực x.

 Bài 28: Tìm tất cả các đa thức P(x) thỏa mãn điều kiện: P(x + 1)= P(x) + 2x +1, với mọi x...

 Bài 29: Tìm tất cả các đa thức P(x) t/m đk: $P((x+1)^{2})=P(x^{2})+2x+1,x\in R.$

 P/s: anh Yolo giải kinh quá :ohmy: full luôn:))

 




#708713 [TOPIC]: ĐA THỨC THCS

Đã gửi bởi PhanThai0301 on 18-05-2018 - 20:41 trong Tài liệu - Đề thi

Bài 4: Khi chia đa thức $x^{1951}-1$ cho đa thức $x^{4}+x^{3}+2x^{2}+x+1$ ta được một thương và phần dư. Hãy tìm hệ số của $x^{14}$ trong thương.
 

 Sau 1 ngày không thấy ai giải mình xin đc trình bày lời giải.

Ta có: $x^{4}+x^{3}+2x^{2}+x+1=(x^{2}+1)(x^{2}+x+1)$=$\frac{x^{12}-1}{x^{8}-x^{7}-x^{6}+2x^{5}-2x^{3}+x^{2}+x-1}$.

            $x^{1951}-1=x^{7}(x^{12}-1)(x^{1932}+x^{1920}+...+x^{12}+1)+x^{7}-1$.

 => $\frac{x^{1951}-1}{x^{12}-1}=x^{1939}+x^{1927}+...+x^{7}+\frac{x^{7}-1}{x^{12}-1}$.

 Suy ra hệ số cần tìm trùng với hệ số của $x^{14}$ trong tích

            $(x^{1939}+...+x^{7}+\frac{x^{7}-1}{x^{12}-1})(x^{8}-x^{7}-x^{6}+2x^{5}-2x^{3}+x^{2}+x+1)$.

  => hệ số này rõ ràng phải bằng -1.




#708683 [TOPIC]: ĐA THỨC THCS

Đã gửi bởi PhanThai0301 on 18-05-2018 - 11:48 trong Tài liệu - Đề thi

Bài 19: (mình tham khảo của duylax2412)Cho đa thức $f(x)=x^n+a_{1}x^{n-1}+a_{2}x^{n-2}+.... 1$ Có các hệ số dương . Chứng  nếu $f(x)$ có $n$ nghiệm thực thì $f(2) \geq 3^n$

 Cảm ơn ĐHV MoMo123 đã tham gia topic của e :wub: mong anh giúp đỡ.

 E xin được giải bài toán tổng quát: Cho n thuộc N* và đa thức P(x)=$x^{n}+a_{1}x^{n-1}+...+a_{n-1}x+1$ với các hệ số không âm $a_{1},a_{2},...,a_{n-1}.$ Biết rằng P(x) có n nghiệm thực. CMR P(m)$\geq (m+1)^{n}$.

 Do $a_{1},a_{2},...,a_{n-1}\geq 0$ nên các nghiệm của P(x) đều ko dương và vì P(0)=1>0 nên các nghiệm của P(x) đều âm. Gọi các nghiệm đó là $-x_{1},-x_{2},...,-x_{n}$ ($x_{i}>0$, với mọi i= 1, 2, 3, ...,n). Khi đó

                          $P(x)=\coprod_{i=1}^{n}(x+x_{i})=x^{n}+a_{1}x^{n-1}+...+a_{n-1}x+1$ nên $x_{1}x_{2}.....x_{n}=1$.

 Áp dụng BĐT Cauchy ta có: 

                          $m+x_{i}=1+1+1...+1(m số 1)+x_{i}\geq (m+1)\sqrt[m+1]{x_{i}}$

                    => $P(m)=\prod_{i=1}^{n}(m+x_{i})\geq (m+1)^{n}.\sqrt[m+1]{x_{1}x_{2}...x_{n}}=(m+1)^{n.}$  

 




#708654 [TOPIC]: ĐA THỨC THCS

Đã gửi bởi PhanThai0301 on 17-05-2018 - 22:45 trong Tài liệu - Đề thi

Bài 6:

Dễ biến đổi P(x) và Q(x) thành:

P(x)=$\frac{(1-x^n)(1+x^n)}{(1-x)(1+x)}$

 

Q(x)=$\frac{1-x^n}{1-x}$

 

Để P(x) chia hết cho Q(x) <=> $(1-x^n)$ chia hết cho 1+x

 

$\frac{x^n+1}{x+1}=H(x)+\frac{1+(-1)^n}{x+1}$

=> $(1-x^n)\vdots (1+x)\Leftrightarrow 1+(-1)^n=0\Leftrightarrow n=2k+1 (k\epsilon N)$

 

 Bạn giải thích rõ đoạn mình bôi xanh được không ?

 

Bài 8: $P(c)=c^2-(a+b)c+ab=c^2 \Rightarrow ab=(a+b)c$

Vì a,b,c nguyên tố cùng nhau nên điều trên xảy ra khi và chỉ khi trong a,b,c có 2 số 0

Khi đó  c.P(a+b)=$ ((a+b)^2-(a+b)(a+b)+ab)c=abc=0$ suy ra đpcm

  Dòng này chưa đúng bạn à!

  Mình xin đc trình bày lời giải của mình

 Ta có: P(c)= $c^{2}-(a+b)c+ab=c^{2}$ => (a-c)(b-c)=$c^{2}$.

 Gọi (a-c, b-c)=d thì $(a-c)\vdots d;(b-c)\vdots d$ =>$c^{2}\vdots d^{2}$=> c$\vdots d$=>$a\vdots d,b\vdots d$

 => d=1 do a, b, c nguyên tố cùng nhau.

 => a-c, b-c đêu là các số chính phương.

 Đặt $a-c=t^{2};b-c=k^{2}=>c=tk$.

 Ta có: a+b=a-c+b-c+2c=$t^{2}+k^{2}+2tk=(t+k)^{2}$.

 => a+b là số chính phương.

 => c.P(a+b)=$c^{2}(a+b)$ là số chính phương (đpcm).

 

Mình xin đưc đ ngh thêm my bài đ mi ngưi luyn tp

Bài 9: Cho đa thc f(x) tha mãn $f(x^{2}-1)=x^{4}-3x^{2}+3$, đúng vi mi x. Tìm $f(x^{2}+1)$.

 

Bài 10: Tìm các h s $b, c$ ca đa thc $P(x)=x^{2}+bx+c$ biết $P(x)$ có giá tr nh nht bng $-1$ khi $x=2.$

 

Bài 11: Cho các đa th$P(x)=x^{3}+ax^{2}+bx+c;Q(x)=x^{2}+2016x+2017$ tha mãn P(x) = 0 có ba nghim thc phân bit và  $P\left ( Q(x) \right )=0$ vô nghim. Chng minh rng $P(2017)> 1008^{6}$.

 

Bài 12: Chng minh rng vi mi s nguyên k, đa thc sau không th có hai nghim nguyên phân bit $P(x)=x^{4}-21x^{3}+(2016+k)x^{2}-2017x+3k$.

 

Bài 13: Cho các đa thc $P(x)$ và $Q(x)$ tha mãn $P(x)=Q(x)+(x^2-x+1).Q(1-x)$ vi mi $ x \in \mathbb{R}$. Biết rng các h s ca $P(x)$ là các s nguyên không âm và $P(0)=$. Tính giá tr $Q(2017)$.

 

Bài 14:  Cho đa thc $P(x)=x^{4}+ax^{3}+bx^{2}+cx+d$ (vi $a,b,c,d$ là các s thc) tha mãn $P(1)=3,P(2)=6,P(3)=11$. Tính $S=10P(4)+P(-2)$.

 

Bài 15: Cho 2 đa thức P(x) và Q(x) thảo mãn P(x)=Q(x) + Q(1-x) với mọi số thực x. Biết rằng các hệ số của đa thức p(x) là các số tự nhiên và P(0)=0. Tính P(P(2017)).




#708646 [TOPIC]: ĐA THỨC THCS

Đã gửi bởi PhanThai0301 on 17-05-2018 - 21:14 trong Tài liệu - Đề thi

$x=0\Rightarrow|c|\leq h\Rightarrow -h\leq c\leq h$

$x=-1\Rightarrow |a-b+c|\leq h\Rightarrow -h\leq a-b+c\leq h$

$x=1\Rightarrow |a+b+c|\leq h\Rightarrow -h\leq a+b+c\leq h$

Từ 3 BĐT trên ta có: $\left\{\begin{matrix} a\leq 2h\\ b,c\leq h \end{matrix}\right. \Rightarrow |a|+|b|+|c|\leq 4h$

 Lời giải rất chuẩn :like mình mới nghĩ ra 1 cách khác sau đây thấy hơi ảo: :wacko:

 Theo giả thiết, ta có $\left | f(0) \right |\leq h,\left | f(1) \right |\leq h,\left | f(-1) \right |\leq h$.

 Ta biểu diễn a, b, c qua f(0), f(1) và f(-1). Ta có

        $\left\{\begin{matrix} f(0)=c & \\ f(1)=a+b+c & \\ f(-1)=a-b+c & \end{matrix}\right.$ hay $\left\{\begin{matrix} c=f(0) & \\a=\frac{f(1)+f(-1)}{2} -f(0) & \\ b=\frac{f(1)-f(-1)}{2} & \end{matrix}\right.$

  Do đó $\left | a \right |+\left | b \right |+\left | c \right |=\left | \frac{f(1+f(-1))}{2} -f(0)\right |+\left | \frac{f(1)-f(-1)}{2} \right |+\left | f(0) \right |\leq \frac{1}{2}\left | f(1) \right |+\frac{1}{2}\left | f(-1) \right |+\left | f(0) \right |+\frac{1}{2}\left | f(1) \right |+\frac{1}{2}\left | f(-1) \right |+\left | f(0) \right |\leq \frac{h}{2}+\frac{h}{2}+h+\frac{h}{2}+\frac{h}{2}+h=4h$ (đpcm)

P/s: mong bạn leuleudoremon giải bài trích lại đề mình cảm ơn.




#708638 [TOPIC]: ĐA THỨC THCS

Đã gửi bởi PhanThai0301 on 17-05-2018 - 20:28 trong Tài liệu - Đề thi

$f(x+1)=x^{2}-3x+2=(x-2)(x-1)=(x+1-3)(x+1-2)$

=>$f(x)=(x-3)(x-2)=x^2-5x+6$

 -> Một cách khá hay mình xin được trình bày cách khác .

  Đặt y = x + 1 thì x = y - 1 và f(x) = $x^{2}-3x+2$ trở thành

         f(y) = $(y-1)^{2}-3(y-1)+2=y^{2}-5y+6$.

 Vì vậy f(x) = $x^{2}-5x+6$.

 




#708636 [TOPIC]: ĐA THỨC THCS

Đã gửi bởi PhanThai0301 on 17-05-2018 - 20:21 trong Tài liệu - Đề thi

$\boxed{\text{Bài 7}}$ Tìm số tự nhiên n sao cho A=$x^{2n}+x^n+1$ chia hết cho $x^2+x+1$ 

 

 Ta xét 3 TH sau:

  •  TH1: n=3k (k là số tự nhiên) thì $x^{2n}+x^{n}+1=x^{6k}+x^{3k}+1=(x^{6k}-1)+(x^{3k}-1)+3$.

           => A chia cho $n^{2}+n+1$ dư 3.

  •  TH2: n=3k+1 thì $x^{2n}+x^{n}+1=x^{6k+2}+x^{3k+1}+1=(x^{6k}-x^{2})+(x^{3k+1}-x)+x^2+x+1$.

           => A chia hết cho $n^{2}+n+1$.

  •  TH3: n=3k+2 thì chứng minh tương tự ta cũng có A chia hết cho $n^{2}+n+1$.

   Kết luận: Vậy nếu n không chia hết cho 3 thì A chia hết cho $n^{2}+n+1$.




#708623 [TOPIC]: ĐA THỨC THCS

Đã gửi bởi PhanThai0301 on 17-05-2018 - 18:00 trong Tài liệu - Đề thi

Mình xin bắt đàu các bài toán sau:

Bài 1: CMR đa thức $x^{9999}+x^{8888}+x^{7777}+x^{6666}+x^{5555}+x^{4444}+x^{3333}+x^{2222}+x^{1111}+1$ chia hết cho đa thức $x^{9}+x^{8}+x^{7}+x^{6}+x^{5}+x^{4}+x^{3}+x^{2}+x+1$.

 

Bài 2: Xác định đa thức f(x) biết rằng với mọi x thì $f(x+1)=x^{2}-3x+2$.

 

Bài 3: Xác định các số thực p, q sao cho đa thức $x^{4}+1$ chia hết cho đa thức $x^{2}+px+q$.

 

Bài 4: Khi chia đa thức $x^{1951}-1$ cho đa thức $x^{4}+x^{3}+2x^{2}+x+1$ ta được một thương và phần dư. Hãy tìm hệ số của $x^{14}$ trong thương.

 

Bài 5: Cho đa thức f(x)=$ax^{2}+bx+c$. CMR nếu $\left | f(x) \right |\leq h$, với mọi $x\in [-1;1]$, thì:

                                                $\left | a \right |+\left | b \right |+\left | c \right |\leq 4h.$




#708618 [TOPIC]: ĐA THỨC THCS

Đã gửi bởi PhanThai0301 on 17-05-2018 - 17:45 trong Tài liệu - Đề thi

   Xin chào tất cả các bạn mình là PhanThai0301 . Mình thấy các bàn toán về đa thức  THCS trên diễn đàn ta có vẻ chưa nhiều lắm nên mình xin đươc lập topic này.

  

   Nội quy của topic:

 

 + Không post những thứ làm spam topic.

 + Post bài cũng như giải bài cần trình bày rõ ràng, cẩn thận.

 + Không nên dẫn link các bài toán đã được giải ở nơi khác.

 + Nếu trong 1 ngày không có ai giải thì post đáp án.

 + Bài nào có lời giải tô màu đỏ, chưa giải tô màu đen.

   Mong mọi người chấp hành đúng nội quy và giải bài hăng say.

   P/s: rất mong được sự góp ý của các anh/ chị trên diễn đàn :D .

  Một số kiến thức và phần đa thức

  1. Định nghĩa

  Đa thức là 1 hàm số P: R -> R có dạng

      P(x)= $a_{n}x^{n}+a_{n-1}x^{n-1}+...+a_{1}x+a_{0}$,

  với $a_{0}, a_{1},...,a_{n}$ là những só thực đã cho, gọi là các hệ số của ẩn x.

  2. Các định lí cơ bản về nghiệm của đa thức và phương pháp để giải các bài toán này

  • Định lí Bezout: Đa thức P(x) chia hết cho đa thức x - c khi và chỉ khi P(c)=0.
  • Phương pháp hệ số bất định: Giả sử f(x)= $a_{3}x^{3}+a_{2}x^{2}+a_{1}x+a_{0}$

                                                                   g(x)= $b_{3}x^{3}+b_{2}x^{2}+b_{1}x+b_{0}$

        Nếu f(x) = g(x) với ít nhất 4 giá trị phân biệt của x thì $a_{3}=b_{3};a_{2}=b_{2};a_{1}=b_{1};a_{0}=b_{0}$.

  • Phương pháp nội suy Newton: Để tìm đa thức P(x) bậc không quá n khi biết giá trị của đa thức tại n+1 điểm: $C_{1},C_{2},...,C_{n+1}$ ta có thể biểu diễn P(x) dưới dạng:

           P(x)= $b_{0}+b_{1}(x-C_{1})+b_{2}(x-C_{1})(x-C_{2})+...+b_{n}(x-C_{1})...(x-C_{n})$

        Bằng cách thế x lần lượt bằng các giá trị của $C_{1},C_{2},...,C_{n+1}$  vào biểu thức P(x) ta có thể tính lần lượt các hệ số của nó.




#708616 Các số thực không âm $x_{1}$,$x_{2}$,...

Đã gửi bởi PhanThai0301 on 17-05-2018 - 17:20 trong Đại số

Các số thực không âm $x_{1}$,$x_{2}$,...,$x_{9}$ thỏa mãn

$\left\{\begin{matrix} x_{1}+x_{2}+...+x_{9}=10\\ x_{1}+2x_{2}+...+9x_{9}=18 \end{matrix}\right.$

Chứng minh: $1.19x_{1}+2.18x_{2}+...+9.11x_{9}\geq 270$ và dấu $=$ xảy ra khi nào?

 

BĐT $\Leftrightarrow 19x_1+36x_2+51x_3+64x_4+75x_5+84x_6+91x_7+96x_8+99x_9\geq 270$

$\Leftrightarrow 17x_2+32x_3 +45x_4+56x_5+65x_6+75x_8+80x_9\geq 80$ 

Từ giả thiết $\Rightarrow x_2+...+8x_9=8$

$\Rightarrow 17x_2+32x_3 +45x_4+56x_5+65x_6+75x_8+80x_9\geq 10x_2+...80x_9=80 (đpcm)$.




#708551 $2^{x}.3^{y}+5^{z}=7^{t}$

Đã gửi bởi PhanThai0301 on 16-05-2018 - 19:46 trong Số học

Giải phương trình nghiệm nguyên dương: $2^{x}.3^{y}+5^{z}=7^{t}$

 Do $x,y,z,t$ nguyên dương nên $2^x.3^y \vdots 3, 7^t \equiv 1 \pmod{3} \Rightarrow 5^z \equiv 1 \pmod{3} \rightarrow z=2k$
TH1: $x\geq 2 \rightarrow 2^x.3^y \vdots 4, 5^z \equiv 1 \pmod{4} \Rightarrow 7^t \equiv 1 \pmod{4} \Rightarrow t=2q$
Khi ấy $2^x.3^y=(7^q-5^k)(7^q+5^k)$
Ta thấy $gcd(7^q,5^k)=1 \rightarrow gcd(7^q-5^k,7^q+5^k)=1,2$ nhưng chúng cùng chẵn nên $gcd(7^q-5^k,7^q+5^k)=2$
Điều đó cũng có nghĩa là trong hai số đó chỉ có một số chia hết cho $2^{x-1}$ còn số còn lại chia $4$ dư $2$
Và $gcd(7^q-5^k,7^q+5^k)=2$ suy ra trong hai số chỉ có một số chia hết cho $3^y$
Do đó ta xét hiệu $7^q+5^k-(7^q-5^k)=2.5^k$
Và hiệu đó chỉ nhận một trong các giá trị sau $|3^y.2^{x-1}-2|,|3^y.2-2^{x-1}|$
Nếu $2.5^k=|3^y.2^{x-1}-2|$ khi ấy dễ thấy $y,x$ nguyên dương nên $3^y.2^{x-1}>2 \Rightarrow 2.5^k=3^y.2^{x-1}-2$ và ngoài ra $2.5^k$ chẵn nên $x-1>0 \rightarrow x\geq 2$
$\Rightarrow 5^k=3^y.2^{x-2}-1$
$\Rightarrow 5^k+1=3^y.2^{x-2}$
Thấy $5^k+1 \equiv 2 \pmod{4} \Rightarrow 2^{x-2} \equiv 2 \pmod{4} \Rightarrow x-2=1 \Rightarrow x=3$
Khi ấy $5^k+1=3^y.2$ như vậy $k=1$ thì $y=1$ và từ đó có bộ nghiệm $(x,y,z,t)$ thỏa mãn (các bạn tự tính)
Còn nếu $k>1 \Rightarrow y>1$ ta có $5^k+1 \vdots 3^y$
Khi ấy $k$ lẻ, ta sẽ cm $5^{3^{y-1}}+1 \vdots 3^y$ mà $\not \vdots 3^{y+1}$
Thấy $y=1$ đúng
Giả sử $y=m$ đúng hay $5^{3^{m-1}}+1 \vdots 3^m$ mà $\not \vdots 3^{m+1}$
Ta sẽ cm $y=m+1$ đúng hay $5^{3^{m}}+1 \vdots 3^{m+1}$ mà $\not \vdots 3^{m+2}$
Thật vậy $5^{3^{m}}+1=(5^{3^{m-1}}+1)(5^{2.3^{m-1}}-5^{3^{m-1}}+1)$
Dễ cm $5^{2.3^{m-1}}-5^{3^{m-1}}+1 \vdots 3$ nhưng không $ \vdots 9$ khi ấy kết hợp cả GTQN có ngay đpcm
Như vậy ta có khẳng định $5^{3^{y-1}}+1 \vdots 3^y$ mà $\not \vdots 3^{y+1}$
Mặt khác ta có $5^k+1 \vdots 3^y \Rightarrow 5^{2k}-1 \vdots 3^y$ và ta cũng có từ trên $5^{2.3^{y-1}}-1 \vdots 3^y$ mà $\not \vdots 3^{y+1}$
Khi ấy dễ thấy $2.3^{y-1}$ là số nhỏ nhất thỏa mãn $5^{2.3^{y-1}}-1 \vdots 3^y$ mà $\not \vdots 3^{y+1}$ thật vậy nếu giả sử có số $r$ nhỏ hơn khi ấy $5^r-1 \vdots 3^y \Rightarrow r$ chẵn mà theo tính chất quen thuộc thì $r|2.3^{y-1} \Rightarrow r=2.3^{g-1}$ với $g<y$
Suy ra $5^{2.3^{g-1}}-1 \vdots 3^y$ mà ta đã cm $5^{2.3^{g-1}}-1 \vdots 3^g$ mà $\not \vdots 3^{g+1}$ khi ấy $g<y$ nên vô lý
Do đó $2.3^{y-1}$ là số nhỏ nhất thỏa mãn hay khi ấy theo bổ đề quen thuộc thì $2k \vdots 2.3^{y-1} \Rightarrow k \vdots 3^{y-1}$
Suy ra $5^k+1 \geq 5^{3^{y-1}}+1>3^y.2$ (dễ dàng cm quy nạp) suy ra vô nghiệm
Nếu $2.5^k=|3^y.2-2^{x-1}|$ khi đó ta không xét thế này mà xét $2.7^q=3^y.2+2^{x-1} \Rightarrow 7^q=3^y+2^{x-2} \Rightarrow 2^{x-2} \equiv 1 \pmod{3} \rightarrow x-2$ chẵn
Trở lại $2.5^k=3^y.2-2^{x-1}$ hoặc $2.5^k=2^{x-1}-3^y.2$
Khi $2.5^k=3^y-2^{x-1} \Rightarrow 5^k+2^{x-2}=3^y$ nếu $x-2=0$ thì $5^k+1=3^y$ thì lại cm như trên $k\geq 3^{y-1}$ khi ấy $5^{k}+1>3^y$ vô lý
Do đó $x-2>0$ mà đã cm $x-2$ chẵn nên $x-2\geq 2$ khi ấy $5^k+2^{x-2} \equiv 1 \pmod{4} \rightarrow 3^y \equiv 1 \pmod{4} \rightarrow y$ chẵn do đó $x-2=2u,y=2v$ khi ấy $5^k=(3^v-2^u)(3^v+2^u)$
$\rightarrow 3^v+2^u=5^p,3^v-2^u=5^q \Rightarrow 5^p-5^q=2^{u+1} \Rightarrow 5^q(5^{p-q}-1)=2^{u+1} \rightarrow q=1$ khi ấy $5^{p-q}-1=2^{u+1}$ bằng chứng minh quy nạp tương tự như trên ta có được $5^{2^{u-1}}-1 \vdots 2^{u+1}$ mà không chia hết cho $2^{u+2}$ và khi đó cũng cm tương tự là $2^{u-1}$ là số nhỏ nhất thỏa mãn khi ấy $p-q \vdots 2^{u-1} \Rightarrow 5^{p-q}-1\geq 5^{2^{u-1}}-1\geq 2^{u+1}$ và dấu $=$ khi $p-q=1,u=2$ và kể từ đó thì $5^{2^{u-1}}-1>2^{u+1}$ (cm quy nạp)
Khi $2.5^k=2^{x-1}-2.3^y \rightarrow 5^k+3^y=2^{x-2}$ và thấy ở trên đã cm được rằng $x-2$ chẵn khi đó $2^{x-2} \equiv 1 \pmod{3} \rightarrow 5^k \equiv 1 \pmod{3} \Rightarrow k$ chẵn khi đó $k=2c,x-2=2d \rightarrow 3^y=(2^d-5^c)(2^d+5^c)$
Do đó $2^d-5^c=3^h,2^d+5^c=3^k \Rightarrow 2^{d+1}=3^k-3^h \Rightarrow h=0 \rightarrow 2^{d+1}=3^k-1$ suy ra $2^{d+1}+1 \vdots 3^k$ và chứng minh tương tự trên có $2^{d+1}+1\geq 2^{3^{k-1}}+1\geq 3^k$ và dấu $=$ khi $k=1,2$ và kể sau đó thì dấu $=$ không xảy ra hay BDT lớn thực sự (cm quy nạp)
Như vậy ta xử lý xong phần này
TH2: $x=1 \rightarrow 2.3^y+5^z=7^t \Rightarrow 5^z \equiv 1 \pmod{3} \Rightarrow z=2r$ khi ấy $5^z \equiv 1 \pmod{8}$
Mặt khác dễ thấy $t$ lẻ vì nếu $t$ chẵn thì $7^t-5^z \vdots 4$ trong khi $2.3^y \equiv 2 \pmod{4}$ vô lý
Như vậy $t$ lẻ và khi đó $7^t-5^q \equiv -1-1 \equiv -2 \pmod{8} \Rightarrow 2.3^y \equiv -2 \pmod{8} \Rightarrow 3^y \equiv -1 \pmod{8} \rightarrow y$ lẻ
Mặt khác ta có $7^t$ lẻ nên $t=4k+1,4k+3$
Nếu $t=4k+1 \rightarrow 7^t \equiv 2 \pmod{5} \Rightarrow 3^y \equiv 1 \pmod{5}$ suy ra $y=4k$ chẵn vô lý với đã cm $y$ lẻ
Nếu $t=4k+3 \rightarrow 7^t \equiv 3 \pmod{5} \Rightarrow 3^y \equiv 4 \pmod{5}$ suy ra $y=4k+2$ chẵn vô lý đã cm $y$ lẻ.

P/s: có cách nào ngắn hơn ko mọi người.




#708545 Cho a, b ,c ,d >0, tìm GTNN của P= \frac{a}{b+2c+3d...

Đã gửi bởi PhanThai0301 on 16-05-2018 - 18:38 trong Bất đẳng thức và cực trị

Lời giải tại đây

Bạn có cách khác sử dựng BDT Bunhiacopski ko?